Difference between revisions of "2003 AIME II Problems/Problem 14"

Line 7: Line 7:
  
 
Letting <math>F = (f,4)</math>, and knowing that <math>\angle FAB = 120^\circ</math>, we can use rewrite <math>F</math> using complex numbers:
 
Letting <math>F = (f,4)</math>, and knowing that <math>\angle FAB = 120^\circ</math>, we can use rewrite <math>F</math> using complex numbers:
<math>f + 4 i = (b + 2 i)\left(e^{(2 \pi / 3)}\right) = (b + 2 i)\left(-1/2 + \frac{\sqrt{3}}{2} i\right) = -\frac{b}{2}-\sqrt{3}+\left(\frac{b\sqrt{3}}{2}-1\right)i.  We solve for </math>b<math> and </math>f<math> and find that </math>F = \left(\frac{8}{\sqrt{3}}, 4\right)<math> and that </math>B = \left(\frac{10}{\sqrt{3}}, 2\right)<math>.
+
<math>f + 4 i = (b + 2 i)\left(e^{(2 \pi / 3)}\right) = (b + 2 i)\left(-1/2 + \frac{\sqrt{3}}{2} i\right) = -\frac{b}{2}-\sqrt{3}+\left(\frac{b\sqrt{3}}{2}-1\right)i</math>.  We solve for <math>b</math> and <math>f</math> and find that <math>F = \left(\frac{8}{\sqrt{3}}, 4\right)</math> and that <math>B = \left(\frac{10}{\sqrt{3}}, 2\right)</math>.
  
The area of the hexagon can then be found as the sum of the areas of two congruent triangles (</math>EFA<math> and </math>BCD<math>, with height </math>8<math> and base </math>\frac{8}{\sqrt{3}}<math> and a parallelogram (</math>ABDE<math>, with height </math>8<math> and base </math>\frac{10}{\sqrt{3}}<math>.
+
The area of the hexagon can then be found as the sum of the areas of two congruent triangles (<math>EFA</math> and <math>BCD</math>, with height <math>8</math> and base <math>\frac{8}{\sqrt{3}}</math> and a parallelogram (<math>ABDE</math>, with height <math>8</math> and base <math>\frac{10}{\sqrt{3}}</math>.
  
</math>A = 2 \times \frac{1}{2} \times 8 \times \frac{8}{\sqrt{3}} + 8 \times \frac{10}{\sqrt{3}} = \frac{144}{\sqrt{3}} = 48\sqrt{3}.
+
<math>A = 2 \times \frac{1}{2} \times 8 \times \frac{8}{\sqrt{3}} + 8 \times \frac{10}{\sqrt{3}} = \frac{144}{\sqrt{3}} = 48\sqrt{3}.
  
Thus, <math>m+n = \boxed{51}</math>.
+
Thus, </math>m+n = \boxed{51}<math>.
  
 
== Solution (Incomplete/incorrect) ==
 
== Solution (Incomplete/incorrect) ==
Line 22: Line 22:
 
{{image}}
 
{{image}}
  
In this image, we have drawn perpendiculars to the <math>x</math>-axis from F and B, and have labeled the angle between the <math>x</math>-axis and segment <math>AB</math> <math>x</math>. Thus, the angle between the <math>x</math>-axis and segment <math>AF</math> is <math>60-x</math> so, <math>\sin{(60-x)}=2\sin{x}</math>. Expanding, we have
+
In this image, we have drawn perpendiculars to the </math>x<math>-axis from F and B, and have labeled the angle between the </math>x<math>-axis and segment </math>AB<math> </math>x<math>. Thus, the angle between the </math>x<math>-axis and segment </math>AF<math> is </math>60-x<math> so, </math>\sin{(60-x)}=2\sin{x}<math>. Expanding, we have
  
<center><math>\sin{60}\cos{x}-\cos{60}\sin{x}=\frac{\sqrt{3}\cos{x}}{2}-\frac{\sin{x}}{2}=2\sin{x}</math></center>
+
<center></math>\sin{60}\cos{x}-\cos{60}\sin{x}=\frac{\sqrt{3}\cos{x}}{2}-\frac{\sin{x}}{2}=2\sin{x}<math></center>
  
Isolating <math>\sin{x}</math> we see that <math>\frac{\sqrt{3}\cos{x}}{2}=\frac{5\sin{x}}{2}</math>, or <math>\cos{x}=\frac{5}{\sqrt{3}}\sin{x}</math>. Using the fact that <math>\sin^2{x}+\cos^2{x}=1</math>, we have <math>\frac{28}{3}\sin^2{x}=1</math>, or <math>\sin{x}=\sqrt{\frac{3}{28}}</math>. Letting the side length of the hexagon be <math>y</math>, we have <math>\frac{2}{y}=\sqrt{\frac{3}{28}}</math>. After simplification we see that <math>y=\frac{4\sqrt{21}}{3}</math>.  
+
Isolating </math>\sin{x}<math> we see that </math>\frac{\sqrt{3}\cos{x}}{2}=\frac{5\sin{x}}{2}<math>, or </math>\cos{x}=\frac{5}{\sqrt{3}}\sin{x}<math>. Using the fact that </math>\sin^2{x}+\cos^2{x}=1<math>, we have </math>\frac{28}{3}\sin^2{x}=1<math>, or </math>\sin{x}=\sqrt{\frac{3}{28}}<math>. Letting the side length of the hexagon be </math>y<math>, we have </math>\frac{2}{y}=\sqrt{\frac{3}{28}}<math>. After simplification we see that </math>y=\frac{4\sqrt{21}}{3}<math>.  
  
 
'''The following is incorrect as the hexagon is NOT regular (although it is equilateral).  The previous work IS correct, so I am leaving it as part of an incomplete solution'''
 
'''The following is incorrect as the hexagon is NOT regular (although it is equilateral).  The previous work IS correct, so I am leaving it as part of an incomplete solution'''
  
The area of the hexagon is <math>\frac{3y^2\sqrt{3}}{2}</math>, so the area of the hexagon is <math>\frac{3*\frac{16*21}{3^2}*\sqrt{3}}{2}=56\sqrt{3}</math>, or <math>m+n=\boxed{059}</math>.
+
The area of the hexagon is </math>\frac{3y^2\sqrt{3}}{2}<math>, so the area of the hexagon is </math>\frac{3*\frac{16*21}{3^2}*\sqrt{3}}{2}=56\sqrt{3}<math>, or </math>m+n=\boxed{059}$.
  
 
== See also ==
 
== See also ==
 
{{AIME box|year=2003|n=II|num-b=13|num-a=15}}
 
{{AIME box|year=2003|n=II|num-b=13|num-a=15}}
 
[[Category:Intermediate Geometry Problems]]
 
[[Category:Intermediate Geometry Problems]]

Revision as of 16:28, 5 January 2010

Problem

Let $A = (0,0)$ and $B = (b,2)$ be points on the coordinate plane. Let $ABCDEF$ be a convex equilateral hexagon such that $\angle FAB = 120^\circ,$ $\overline{AB}\parallel \overline{DE},$ $\overline{BC}\parallel \overline{EF,}$ $\overline{CD}\parallel \overline{FA},$ and the y-coordinates of its vertices are distinct elements of the set $\{0,2,4,6,8,10\}.$ The area of the hexagon can be written in the form $m\sqrt {n},$ where $m$ and $n$ are positive integers and n is not divisible by the square of any prime. Find $m + n.$

Solution

The y-coordinate of $F$ must be $4$. All other cases yield non-convex hexagons, which violate the problem statement.

Letting $F = (f,4)$, and knowing that $\angle FAB = 120^\circ$, we can use rewrite $F$ using complex numbers: $f + 4 i = (b + 2 i)\left(e^{(2 \pi / 3)}\right) = (b + 2 i)\left(-1/2 + \frac{\sqrt{3}}{2} i\right) = -\frac{b}{2}-\sqrt{3}+\left(\frac{b\sqrt{3}}{2}-1\right)i$. We solve for $b$ and $f$ and find that $F = \left(\frac{8}{\sqrt{3}}, 4\right)$ and that $B = \left(\frac{10}{\sqrt{3}}, 2\right)$.

The area of the hexagon can then be found as the sum of the areas of two congruent triangles ($EFA$ and $BCD$, with height $8$ and base $\frac{8}{\sqrt{3}}$ and a parallelogram ($ABDE$, with height $8$ and base $\frac{10}{\sqrt{3}}$.

$A = 2 \times \frac{1}{2} \times 8 \times \frac{8}{\sqrt{3}} + 8 \times \frac{10}{\sqrt{3}} = \frac{144}{\sqrt{3}} = 48\sqrt{3}.

Thus,$ (Error compiling LaTeX. Unknown error_msg)m+n = \boxed{51}$.

== Solution (Incomplete/incorrect) == {{image}}

From this image, we can see that the y-coordinate of F is 4, and from this, we can gather that the coordinates of E, D, and C, respectively, are 8, 10, and 6.

{{image}}

In this image, we have drawn perpendiculars to the$ (Error compiling LaTeX. Unknown error_msg)x$-axis from F and B, and have labeled the angle between the$x$-axis and segment$AB$$ (Error compiling LaTeX. Unknown error_msg)x$. Thus, the angle between the$x$-axis and segment$AF$is$60-x$so,$\sin{(60-x)}=2\sin{x}$. Expanding, we have

<center>$ (Error compiling LaTeX. Unknown error_msg)\sin{60}\cos{x}-\cos{60}\sin{x}=\frac{\sqrt{3}\cos{x}}{2}-\frac{\sin{x}}{2}=2\sin{x}$</center>

Isolating$ (Error compiling LaTeX. Unknown error_msg)\sin{x}$we see that$\frac{\sqrt{3}\cos{x}}{2}=\frac{5\sin{x}}{2}$, or$\cos{x}=\frac{5}{\sqrt{3}}\sin{x}$. Using the fact that$\sin^2{x}+\cos^2{x}=1$, we have$\frac{28}{3}\sin^2{x}=1$, or$\sin{x}=\sqrt{\frac{3}{28}}$. Letting the side length of the hexagon be$y$, we have$\frac{2}{y}=\sqrt{\frac{3}{28}}$. After simplification we see that$y=\frac{4\sqrt{21}}{3}$.

'''The following is incorrect as the hexagon is NOT regular (although it is equilateral). The previous work IS correct, so I am leaving it as part of an incomplete solution'''

The area of the hexagon is$ (Error compiling LaTeX. Unknown error_msg)\frac{3y^2\sqrt{3}}{2}$, so the area of the hexagon is$\frac{3*\frac{16*21}{3^2}*\sqrt{3}}{2}=56\sqrt{3}$, or$m+n=\boxed{059}$.

See also

2003 AIME II (ProblemsAnswer KeyResources)
Preceded by
Problem 13
Followed by
Problem 15
1 2 3 4 5 6 7 8 9 10 11 12 13 14 15
All AIME Problems and Solutions